Author Archives: Hung Nguyen

Góc định hướng và ứng dụng

Góc định hướng. 

Góc giữa hai tia. Cho hai tia $Ox, Oy$, ta cho tia $Ot$ lúc đầu trùng với $Ox$ và cho $Ot$ quay quanh $O$, đến khi $Ot$ trùng với $Oy$, ta nó $Ot$ tạo ra một góc lượng giác (góc định hướng) có tia đầu là $Ox$ tia cuối là $Oy$, kí hiệu $(Ox, Oy)$.

Chú ý: Với hai tia $Ox, Oy$ thì có vô số góc lượng giác có tia đầu $Ox$ tia cuối $Oy$ và hơn kém nhau $k2 \pi$.

Góc giữa hai đường thẳng. Cho hai đường thẳng $a, b$ cắt nhau tại $O$, ta cho đường thằng $t$ qua $O$ lúc đầu trùng với $O$, quay $t$ quanh $O$ đến khi $t$ trùng $a$, ta nói $t$ tạo ra góc giữa đường thẳng $a, b$, kí hiệu là $(a;b)$.

Các góc lượng giác tạo giữa $a, b$ hơn kém nhau $k\pi$.

Một số tính chất thường sử dụng. 

Tính chất 1. Hệ thức Charles
a) Cho $a, b, c$ là ba đường thẳng bất kì thì $(a, b)=(a, c)+(c, b)(\bmod \pi)$
b) $\mathrm{Cho} O x, O y, O z$ là ba tia thì $(O x, O y)=(O x, O z)+(O z, O y)(\bmod 2 \pi)$

Tính chất 2. (Điều kiện 3 điểm thẳng hàng) Cho 3 điểm $A, B, C$ và đường thẳng $d$. Khi đó $A, B, C$ thẳng hàng khi và chỉ khi $(A B, d)=(A C, d)(\bmod \pi)$

Tính chất 3. (Điều kiện 4 điểm đồng viên) Cho 4 điểm $A, B, C, D .$ Khi đó $A, B, C, D$ cùng thuộc một đường tròn khi và chỉ khi $(A C, A D)=(B C, B D)(\bmod \pi)$.

Tính chất 4. Nếu $a$ là phân giác của góc tạo bởi hai đường thẳng $b, c$ thì $(b, a)=-(c, a)=\frac{1}{2}(b, c)\left(\bmod \frac{\pi}{2}\right)$

Tính chất 5. Nếu $a$ và $a^{\prime}$ đỗi xứng nhau qua đường thẳng $d$ thì $(a, d)=-\left(a^{\prime}, d\right)(\bmod \pi)$.

Tính chất 6 . Nếu $a^{\prime}$ là ảnh của $a$ qua phép quay với góc quay $\alpha$ thì $\left(a, a^{\prime}\right)=\alpha(\bmod \pi)$

Các ví dụ

Bài 1. (Định lý Migel) Cho tam giác $A B C$; Gọi $D, E, F$ lần lượt là các điểm thuộc các đường thẳng $B C, A C$ và $A B$
a) Chứng minh rằng các đường tròn ngoại tiếp các tam giác $A E F, B F E, C D E$ cùng đi qua một điểm $M$.
b) Nếu $D, E, F$ thẳng hàng thì điểm $M$ thuộc đường tròn ngoại tiếp của tam giác $A B C$; hơn nữa tâm các đường tròn $(A B C),(A E F),(B F E),(C D E)$ cùng thuộc một đường tròn và đường tròn đó qua $\mathrm{M}$.

Lời giải

a) Gọi $M$ là giao điểm của $(A E F)$ và $(B D F)$, ta chứng minh $C, D, E, M$ đồng viên.
Ta có $(E M ; E C)=(E M ; E A)=(F M ; F A)(\bmod \pi)($ Do $A, E, M, F$ đồng viên $)$
Mà $(F M, F A)=(F M ; F B)=(D M: D B)(\bmod \pi)($ Do $D, M, F, B$ đồng viên $)$
Suyra $(E M ; E C)=(D M ; D B)=(D M ; D C)(\bmod \pi)$
Do đó $M, E, C, D$ đồng viên.

b) $\operatorname{Tacó}(A M: A F)=(E M ; E F)(\bmod \pi),(A M: A F)=(C M ; C B)(\bmod \pi)$ và $(C M ; C B)=(E M ; E D)(\bmod \pi)$
Do đó $E, D, F$ thẳng hàng khi và chỉ khi $(E M ; E F)=(E M: E D)$ khi và chỉ khi $(A M ; A F)=(C M ; C B)$ khi và chỉ khi $A \cdot B, C, M$ đồng viên.
Gọi $O, O_{a}, O_{b}, O_{c}$ lần lượt là tâm đường tròn ngoại tiếp các tam giác $A B C, A E F, B D F, C D E$. Ta chứng minh $O, M, O_{a}, O_{b}, O_{c}$ đồng viên.
Thật vậy ta có $\left(O_{a} M ; O_{a} O_{b}\right)=(E M ; E F)=(C M ; C D)=\left(O M ; O O_{b}\right)(\bmod \pi)$. Do đó $O_{a}, M, O, O_{b}$ đồng viên. Tương tự $O_{a}, M, O, O_{c}$ đồng viên. Suy ra điều cần chứng minh.

Bài 2. (Đường thẳng Steiner – Điểm Antisteiner)

a) Cho tam giác $A B C$ nội tiếp đường tròn $(O), M$ là một điểm thuộc $(O)$. Gọi \$latex $A^{\prime}, B^{\prime}, C^{\prime} \$$ lần lượt là điểm đối xúng của $M$ qua $B C, A C, A B$. Chứng minh rằng $A^{\prime}, B^{\prime}, C^{\prime}$ cùng thuộc một đường thẳng và đường thẳng đó qua trực tâm $\mathrm{H}$ của tam giác $A B C$.
b) Ngược lại lấy $d$ là một đường thẳng qua $H$. Gọi $d_{a}, d_{b}, d_{c}$ lần lưọt là các đường thẳng đối xứng của d qua BC, $A C, A B$. Chúng minh rằng $d_{a}, d_{b}, d_{c}$ đồng qui tai một điểm thuộc đường tròn $(O)$.

Lời giải

a) Gọi $H_{c}, H_{b}$ là điểm đõi xứng của $H$ qua $A B: A C$. Ta có $H_{c}, H_{b} \in(A B C)$
a) $\left(H C^{\prime} ; H B^{\prime}\right)=\left(H C^{\prime} ; H A\right)+\left(H A ; H B^{\prime}\right)=-\left(H_{c} M ; H A\right)-\left(H_{b} A ; H_{b} M\right)=0($ $\bmod \pi)$
Vầy $H, B^{\prime}, C^{\prime}$ thẳng hàng.

b) Ta thấy $H_{a} \in d_{a}, H_{b} \in d_{b} \cdot$ Gọi $M$ là giao điểm của $d_{a}, d_{b}$. Ta chứng minh $M \in(A B C)$. Ta có:
$$
\begin{aligned}
&\left(M H_{a} ; M H_{b}\right)=\left(A^{\prime} H_{a} ; A^{\prime} C\right)+\left(A^{\prime} C ; C A\right)+\left(C A ; M H_{b}\right) \\
&=-\left(A^{\prime} H ; B C\right)+(C B ; C A)-\left(C A ; B^{\prime} H\right) \\
&=\left(B C ; A^{\prime} H\right)+\left(B^{\prime} H ; C A\right)+(C B ; C A) \\
&=2(B C ; C A)(\bmod \pi) \\
&=\left(C H_{a} ; C H_{b}\right)(\bmod \pi)
\end{aligned}
$$
Do đó $M \in(A B C)$.

Bài 3. 
a) Cho tam giác $A B C$ nội tiếp đương tròn $(O), P Q$ là đương kính. Chứng minh rằng đường thẳng Simson của tam giác ABC úng vói các điểm $P, Q$ vuông góc nhau.
b) Tổng quát hơn, nếu $P Q$ là dây cung bất kì thì góc tạo bởi hai đương thẳng Simson ứng với $P$ và $Q$ bằng nủa số đo chung nhỏ $P Q$.

Lời giải

b)
$$
\begin{aligned}
(\mathrm{DI} ; \mathrm{JK}) &=(\mathrm{DI} ; \mathrm{DP})+(\mathrm{DP} ; \mathrm{AC})+(\mathrm{AC} ; \mathrm{CJ})+(\mathrm{CJ} ; \mathrm{JK})(\bmod \pi) \\
&=(\mathrm{CI} ; \mathrm{CP})+1 / 2 \pi+(\mathrm{AC} ; \mathrm{BC})+(\mathrm{QC} ; \mathrm{QK})(\bmod \pi) \\
&=(\mathrm{CB} ; \mathrm{CP})+1 / 2 \pi+(\mathrm{AC} ; \mathrm{BC})+(\mathrm{CQ} ; \mathrm{CK})+(\mathrm{CK} ; \mathrm{QK})(\bmod \pi) \\
&=(\mathrm{CB} ; \mathrm{CP})+(\mathrm{AC} ; \mathrm{CB})+(\mathrm{CQ} ; \mathrm{CA})(\bmod \pi) \\
&=(\mathrm{CQ} ; \mathrm{CP})(\bmod \pi) \square
\end{aligned}
$$

Bài 4. (Chọn đội dự tuyển PTNK 2008) Cho tam giác ABC. Các điểm $M, N, P$ lần luợt thuộc các đt $B C, C A$, AB sao cho tam giác MNP và tam giác $A B C$ đồng dạng. Chúng minh ràng tâm đưòng tròn ngoại tiếp của tam giác $B C$ là thục tâm của tam giác $M N P$.

Lời giải

Theo định lý Migel thì các đường tròn (ANP), (BMP) và (CMN) cắt nhau tại $O$. Ta có
$$
\begin{aligned}
(\overline{O B} ; \overrightarrow{O C}) &=(\overline{O B} ; \overline{O P})+(\overline{O P} ; \overline{O M})+(\overline{O M} ; \overrightarrow{O C}) &(\bmod \pi) \\
&=(\overline{M B} ; \overline{M P})+(\overline{O P} ; \overline{O M})+(\overline{O M} ; \overrightarrow{O C}) &(\bmod \pi) \\
&=-(\overline{M P} ; \overline{M B})+(\overline{B P} ; \overline{B M})+(\overline{N M} ; \overrightarrow{N C}) \quad(\bmod \pi) \\
&=-(\overline{M P} ; \overline{M N})-(\overline{M N} ; \overline{M B})+(\overline{B P} ; \overline{B M})+(\overline{N M} ; \overrightarrow{N C}) \\
&=(\overline{M N} ; \overline{M P})+(\overline{M B} ; \overline{M N})+(\overline{M N} ; \overline{C N})+(\overline{B P} ; \overline{B M}) \\
&=(\overline{M N} ; \overline{M P})+(\overline{M B} ; \overline{C N})+(\overline{B P} ; \overline{B M}) \\
&=(\overline{M N} ; \overline{M P})+(\overline{B P} ; \overline{C N})=2(A B ; A C) \quad(\bmod \pi)
\end{aligned}
$$
Từ đó ta có $\mathrm{O}$ là tâm đường tròn ngoại tiếp tam giác $\mathrm{ABC}$.

Mặt khác
$$
\begin{aligned}
(O N ; P M) &=(O N ; O P)+(O P ; P M) & &(\bmod \pi) \\
&=(A N ; A P)+(B O ; B M) & &(\bmod \pi) \\
&=(A C ; A B)+(B O ; B C) & &(\bmod \pi) \\
&=\frac{\pi}{2} \quad &(\bmod \pi)
\end{aligned}
$$
Suy ra $\mathrm{ON} \perp \mathrm{PM}$. Chứng minh tương tự ta có $\mathrm{MO} \perp \mathrm{NP}$. Hay $\mathrm{O}$ là trực tâm của tam giác $\mathrm{ABC}$.

Bài 5. Cho hai hình vuông $A B C D$ và $A E F G$ cùng hướng, $A, B, E$ không thẳng hàng. Chứng minh rẳng $B E, C F, D G$ đồng quy.

Lời giải

 

Xét phép quay tâm A góc quay $(A B: A D)=90^{\circ}$. Khi đó $B$ biên thành $D, E$ biên thành $G$. Gọi $H$ là giao điểm của $\mathrm{BE}$ và $\mathrm{GD}$. Khi đó $(B E ; G D)=(A B ; A D)=(C B ; C D)=90^{\circ}(\bmod \pi)$. Suy ra $A, H, B, C, D$ đồng viên.
Từ đó ta có $(H B: H C)=(A B: A C)(\bmod \pi)$,
Hơn nữa, $(H G ; H E)=(A G ; A E)=90^{\circ}(\bmod \pi)$ nên $A, E, H, G, F$ cũng đồng viên. Suy ra $(H E ; H F)=(A B: A C)(\bmod \pi)$
Ta có $(H B ; H C)=(H E ; H F)(\bmod \pi)$ mà $H, E, B$ thẳng hàng nên $H, C, F$ thẳng hàng, hay $B E . C F, D G$ đồng quy.

Bài tập rèn luyện

Bài 1 (VMO 2006) Cho tứ giác lồi $A B C D$. Xét một điểm $M$ di động trên đường thẳng $A B$ sao cho $M$ không trùng với $A$ và B. Gọi $N$ là giao điểm thứ hai khác $M$ của đường tròn đi qua 3 điểm $M, N, C$ và đường tròn đi qua 3 điếm $M, B$, D. Chứng minh:
a) Điểm $\mathrm{N}$ di động trên một đường tròn cố định.
b) Đường thẳng MN luôn đi qua một điểm cố định.

Bài 2. Cho tứ giác lồi $A B C D$ nội tiếp một đường tròn. Gọi $P, Q, R, S$ là giao điểm của các đường phân giác ngoài của Các góc ADB và ADB, DAB Và DBA, ACD và ADC, DAC và DCA tương ứng. Chứng minh rẳng $P, Q, R, S$ đồng viên.

Bài 3. Cho tứ giác $A B C$. Chứng minh rằng đường tròn Euler của các tam gíác $A B C, A C D, A B D$ và $B C D$ cùng đi qua một điểm.

Bài 4. Cho hai đường tròn (O) và (O’) cắt nhau tại $A$ và B. Một đường thẳng qua A cắt $(O)$ và $\left(O^{\prime}\right)$ tai $M$ và N. Một đường thẳng qua $B$ cắt $(O)$ và $(O)$ tai $P$ và Q. Chứng minh $M P / / N Q .$

Bài 5. Cho tam giác $A B C$, đưòng cao $A H$ (H thuộc BC). Gọi $D, E$ là hình chiếu của H trên $A B$ và $A D$, đương thẳng $D E$ cắt $B C$ tại $F$. Goi $O_1, O_2$ là tâm đương tròn ngoại tiếp các tam giác BDF và CEF; gọi I là trung điểm $\mathrm{AH}$ và $\mathrm{O}$ là tâm đường tròn ngoại tiếp tam giác $A B C$. Chúng minh rằng 4 điểm $I, O, O_1$ và $O_2$ cùng thuộc một đương tròn.

Giới thiệu sách dành cho học sinh chuyên toán THPT – Đại số

Đại số trong chương trình chuyên toán THPT thường có các nội dung: Phương trình hàm, đa thức, bất đẳng thức, dãy số, … có rất nhiều tài liệu tham khảo tiếng Việt và Tiếng Anh, ở đây xin gởi một số sách cho các bạn tham khảo

Sách tham khảo đại số

Ngoài ra thì có sách bản tiếng Việt bán ngoài thị trường của các tác giả trong nước, không có file pdf, các bạn có thể tìm mua trên các kênh bán sách online.

  1. Các bài toán phương trình hàm trong các kì thi Olympic – Nguyễn Trọng Tuấn
  2. Chuyên đề đa thức bồi dưỡng học sinh giỏi – Nguyễn Tài Chung và đồng nghiệp.
  3. Các sách bất đẳng thức của Võ Quốc Bá Cẩn và đồng nghiệp

Đáp án thi chọn Đội Tuyển thi Quốc Gia của trường PTNK năm học 2015 – 2016

Ngày thứ 1

Bài 1. Cho tập hợp
$$
A=\{n \in \mathbb{N} \mid 1 \leq n \leq 2015,(n, 2016)=1\}
$$
Hỏi có bao nhiêu số nguyên $a \in A$ sao cho tồn tại số nguyên b mà $a+2016 b$ là số chính phương?

Bài 2. Cho $a, b, c, d$ là các số thực thỏa mãn điều kiện
$$
a^{2} \leq 1, a^{2}+b^{2} \leq 5, a^{2}+b^{2}+c^{2} \leq 14, a^{2}+b^{2}+c^{2}+d^{2} \leq 30
$$
1. Chúng minh rằng $a+b+c+d \leq 10$.
2. Chứng minh rằng $a d+b c \leq 10$.

Bài 3. Tìm tất cả các hàm số $f: \mathbb{R} \rightarrow \mathbb{R}$ thỏa mãn điều kiện
$$
f(x-2 f(y))=5 f(x)-4 x-2 f(y)
$$
với mọi $x, y \in \mathbb{R}$.

Bài 4. Cho đường tròn $k$ và các điểm $B, C$ thuộc đường tròn, không phải là đường kính; I là trung điểm $B C$. Điểm $A$ di động trên cung lớn $B C$ của $k$. Gọi $i_{1}$ là đường tròn qua $I$ và tiếp xúc với $A B$ tại $B ; i_{2}$ là đường tròn qua $I$ và tiếp xúc với $A C$ tại $C$. Các đường tròn $i_{1}, i_{2}$ cắt nhau tại $D$ (khác $I$ ).
1. Chứng minh rằng đường tròn ngoại tiếp tam giác AID luôn đi qua một điểm cố định.
2. Gọi $K$ là trung điểm $A D$, $E$ là tâm đường tròn qua $K$ và tiếp xúc với $A B$ tại $A, F$ là tâm đường tròn qua $K$ và tiếp xúc với AC tại $A$. Chứng minh rằng góc EAF có số đo không đổi.

Ngày thứ 2

Bài 5. Dãy số $\left(x_{n}\right)$ được xác định bởi công thức $x_{n}=\frac{1}{n \cos \frac{1}{n}}$ với mọi $n \geq 1$. Tính giới hạn sau
$$\lim \frac{x_{1}+x_{3}+x_{5}+\cdots+x_{2 n-1}}{x_{2}+x_{4}+x_{6}+s+x_{2 n}}$$

Bài 6. Tim các giá trị của $b$ sao cho tồn tại a để hệ phương trình sau có nghiệm $(x, y)$
$$
\left\{\begin{array}{l}
(x-1)^{2}+(y+1)^{2}=b \\y=x^{2}+(2 a+1) x+a^{2}
\end{array}\right.
$$

Bài 7. Cho n là số nguyên dương, $n \geq 2$ và $X={1,2,3, \ldots, n}$. Gọi $A_{1}, A_{2}, \ldots, A_{m}$ và $B_{1}, B_{2}, \ldots, B_{m}$ là hai dãy các tập con khác rỗng của $X$ thỏa mãn điều kiện: Với mỗi $i, j \in{1,2,3, \ldots, n}, A_{i} \cap B_{j}=\varnothing$ nếu và chỉ nếu $i=j$.
1. Chúng minh rằng với mỗi hoán vị $\left(x_{1}, x_{2}, \ldots, x_{n}\right)$ của $X$, có không quá một cặp tập hợp $\left(A_{i}, B_{i}\right)$ với $i=1,2,3, \ldots, n$ sao cho nếu $x_{k} \in A_{i}$ và $x_{l} \in B_{i}$ thì $k<l$.
2. Gọi $a_{i}, b_{i}$ lần lượt là số phần tử của tập hợp $A_{i}, B_{i}$ với $i=1,2,3, \ldots, m$. Chúng minh rằng
$$
\sum_{i=1}^{m} \frac{1}{C_{a_{i}+b_{i}}^{a_{i}}} \leq 1
$$

Bài 8. Cho tam giác $A B C$ nhọn nội tiếp đường tròn tâm $O$. Đường tròn tâm $I$ đi qua $B$, $C$ lần lượt cắt các tia $B A$, CA tại $E, F$.
1. Giả sử các tia $B F, C E$ cắt nhau tại $D$ và $T$ là tâm đường tròn $(A E F)$. Chứng minh rằng $O T$ || ID.
2. Trên BF, CE lần lượt lấy các điểm $G, H$ sao cho $A G \perp C E, A H \perp B F$. Các đường tròn $(A B F),(A C E)$ cắt $B C$ tai $M, N$ (khác $B, C)$ và cắt EF tại $P, Q$ (khác $E, F)$. Gọi $K$ là giao điểm của $M P, N Q$. Chứng minh rằng DK vuông góc với GH.

Giải

Bài 1.

Cho $n$ là số nguyên dương lớn hơn 1 , ta quy ước gọi một số nguyên dương a được gọi là thặng dư chính phương theo modulo $n$ nếu $(a, n)=1$ và tồn tại số nguyên $x$ sao cho $a \equiv x^{2}(\bmod n)$. Trong bài này, dể dơn giản, ta quy ước xét các thặng dư chính phưong nhỏ hơn $n$.
Đặt $s(n)$ là số các số nhỏ hơn $n$ và là thặng dư chính phương theo modulo n. Ta sễ chứng minh hai bổ dề dưới đây:
Bổ đề 1: Cho $p$ là số nguyên tố và $k$ là số nguyên dưong. Khi đó:
1. Nếu $p=2$ thì $s\left(2^{k}\right)=2^{\max (k-3,0)}$.
2. Nếu $p>2$ thì $s\left(p^{k}\right)=\frac{p^{k}-p^{k-1}}{2}$.
Bổ đề $2: s(n)$ là hàm nhân tính.
Thật vậy,
Trước hết, ta biết rằng $s(p)=\frac{p-1}{2}$ với $p$ là số nguyên tố lẻ. Ta sẽ tính $s\left(p^{k}\right)$ với $k \in \mathbb{Z}^{+}$.
Xét một thặng dư chính phương $a$ của $p$, khi đó tồn tại $x$ sao cho
$$
a \equiv x^{2}(\bmod p)
$$
Dặt $a=x^{2}+p q$ thì hiển nhiên
$$
a \equiv x^{2}+p q\left(\bmod p^{k}\right) \Leftrightarrow a-p q \equiv x^{2}\left(\bmod p^{k}\right)
$$
và khi đó, ta có $p^{k-1}$ cách chọn $q$ để các số $a-p q$ là các thặng dư chính phưong $\bmod p^{k}$.
Suy ra
$$
s\left(p^{k}\right)=p^{k-1} s(p)=\frac{p^{k}-p^{k-1}}{2}
$$
Xét số nguyên tố $p=2$, với $k=1,2,3$, dể dàng kiểm tra được $s\left(2^{k}\right)=1$.
Ta xét $k \geq 4$, tưong tự trên, ờ bước chọn $q$, ta chỉ có 2 cách nên $s\left(2^{k}\right)=$ $2 s\left(2^{k-1}\right)$. Từ đó bằng quy nạp, ta có được
$$
s\left(2^{k}\right)=2^{k-3}, k \geq 4
$$
Tiếp theo, xét hai số $a, b$ nguyên dương và $(a, b)=1 .$ Gọi $A$ là tập họp các thặng dư chính phương theo modulo $a b$ và $B$ là tập hợp các số là thặng dư chính phưong chung của $a, b$.
Nếu $x \in A$ thì tổn tại $y$ sao cho $x \equiv y^{2}(\bmod a b)$. Rō ràng khi đó,
$$
x \equiv y^{2} \quad(\bmod a), x \equiv y^{2} \quad(\bmod b)
$$

(chú ý rằng nếu $x>a$, ta có thể chọn $x^{\prime}$ sao cho $x^{\prime}<a$ và $x \equiv x^{\prime}(\bmod a)$; tương tự với $b$ ). Do đó, $x \in B$, tức là $x \in A \Rightarrow x \in B$ nên $|A| \leq|B|$.
Tiếp theo, xét $x \in B$. Khi đó tồn tại $r, s$ sao cho $x \equiv r^{2}(\bmod a), x \equiv s^{2}$ $(\bmod b)$. Theo định lý thặng dư Trung Hoa, tổn tại số nguyên $z$ sao cho
$$
z \equiv r(\bmod a), z \equiv s(\bmod b)
$$
Khi đó
$$
x \equiv z^{2} \quad(\bmod a), x \equiv z^{2} \quad(\bmod b)
$$
nên
$$
x-z^{2}: a b \text { hay } x \equiv z^{2}(\bmod a b)
$$
Do đó: $x \in A$, tức là $x \in B \Rightarrow x \in A$ nên $|A| \geq|B|$.
Từ đây ta có
$$
|A|=|B| \text { hay } s(a) s(b)=s(a b)
$$
Vậy $s(n)$ là hàm nhân tính.
Các bổ đề đều được chứng minh.
Trở lại bài toán, ta thấy rằng
$$
2016=2^{5} \cdot 3^{2} \cdot 7
$$
Rō ràng bài toán yêu cầu đếm số thặng dư chính phương theo modulo 2016. Theo bổ dề 2 thì
$$
s(2016)=s\left(2^{5}\right) s\left(3^{2}\right) s(7)
$$
Theo bổ đề 1 thì
$$
s\left(2^{5}\right)=2^{2}=4, s\left(3^{2}\right)=\frac{3^{2}-3}{2}=3, s(7)=\frac{7-1}{2}=3
$$
Do đó, số các số $a$ cần tìm là $4 \cdot 3 \cdot 3=36$.

Bài 2.

1) Dự đoán dấu bằng xảy ra khi $a=1, b=2, c=3, d=4$ nên ta có các đánh giá sau
$$
\left\{\begin{array}{l}
a^{2}+1 \geq 2 a \\
b^{2}+4 \geq 4 b \\
c^{2}+9 \geq 6 c \\
d^{2}+16 \geq 8 d
\end{array}\right.
$$
Do đó, ta có
$$
\begin{aligned}
&24(a+b+c+d) \leq 3\left(d^{2}+16\right)+4\left(c^{2}+9\right)+6\left(b^{2}+4\right)+12\left(a^{2}+1\right) \\
&=3 d^{2}+4 c^{2}+6 b^{2}+12 a^{2}+120 \\
&=3\left(a^{2}+b^{2}+c^{2}+d^{2}\right)+\left(a^{2}+b^{2}+c^{2}\right)+2\left(a^{2}+b^{2}\right)+6 a^{2}+120 \\
&\leq 3 \cdot 30+14+2 \cdot 5+6 \cdot 1+120=240
\end{aligned}
$$
Suy ra $a+b+c+d \leq 10$.
2) Ta có:
$$
16 a^{2}+d^{2} \geq 8 a d \text { và } 9 b^{2}+4 c^{2} \geq 12 b c
$$
Từ đó suy ra
$$
\begin{aligned}
&24(a d+b c) \leq 3\left(16 a^{2}+d^{2}\right)+2\left(9 b^{2}+4 c^{2}\right) \\
&=3\left(a^{2}+b^{2}+c^{2}+d^{2}\right)+5\left(a^{2}+b^{2}+c^{2}\right)+10\left(a^{2}+b^{2}\right)+30 a^{2} \\
&\leq 3 \cdot 30+5 \cdot 14+10 \cdot 5+30 \cdot 1=240
\end{aligned}
$$
Suy ra $a d+b c \leq 10$.

Bài 3.

Goi $(*)$ là điều kiện đề bài cho. Trong $(*)$, thay $x=y=0$, ta có
$$
f(-2 f(0))=3 f(0)
$$
Đặt $f(0)=a$ thì $f(-2 a)=3 a$. Trong $(*)$, thay $x=0$ và $y=-2 a$, ta có
$$
f(-2 f(-2 a))=5 a-2 f(-2 a) \Leftrightarrow f(-6 a)=-a
$$

Trong $(*)$, thay $x=-2 a, y=-6 a$, ta có
$$
\begin{aligned}
&f(-2 a-2 f(-6 a))=5 f(-2 a)-4 x-2 f(-6 a) \\
&\Leftrightarrow f(0)=15 a+8 a+2 a \\
&\Leftrightarrow a=25 a \\
&\Leftrightarrow a=0
\end{aligned}
$$
Do đó $f(0)=0$.
Trong $(*)$, thay $y=0$, ta có
$$
f(x)=5 f(x)-4 x \Leftrightarrow f(x)=x
$$
Thử lại ta thấy thỏa.
Vậy hàm số cần tìm chính là
$$
f(x)=x, \forall x \in \mathbb{R}
$$

Bài 4.

1) Gọi $O$ là tâm của đường tròn $k$. Không mât tính tống quát, giả sử tia $\Lambda D$ nằm giữa hai tia $A O, A B$, các trường hợp còn lại tương tự.
Ta có:
$$
\angle I D B=\angle A B C, \angle I D C=\angle A C B
$$
nên
$$
\angle B A C+\angle B D C=\angle B A C+\angle A B C+\angle A C B=180^{\circ}
$$

Do đó, tứ giác $A B D C$ nội tiếp hay $D \in(O)$. Ta thấy
$$
\begin{aligned}
&\angle D A O+\angle O I D \\
&=\angle B A C-(\angle D A B+\angle O A C)+360^{\circ}-\left(90^{\circ}+\angle D I C\right) \\
&=\angle B A C-\left(\angle I C D+90^{\circ}-\angle A B C\right)+270^{\circ}-\angle D I C \\
&=\angle B A C+\angle A B C-(\angle I C D+\angle D I C)+180^{\circ} \\
&=\left(180^{\circ}-\angle A C B\right)-\left(180^{\circ}-\angle I D C\right)+180^{\circ} \\
&=\angle I D C-\angle A C B+180^{\circ}=180^{\circ}
\end{aligned}
$$

Do đó, AOID nội tiếp hay đường tròn $(A I D)$ di qua $O$ cố định.
2) Ta có:
$$
\angle E A C=90^{\circ}-\angle B A C, \angle F A B=90^{\circ}-\angle B A C
$$
nên
$$
\angle E A F=180^{\circ}-2 \angle B A C+\angle B A C=180^{\circ}-\angle B A C
$$
Do đó, góc $\angle E A F$ có số đo không đổi.

Bài 5.

Trước hết, ta chứng minh bổ đề sau:
Giá trị của biểu thức
$$
\frac{1}{1}+\frac{1}{2}+\frac{1}{3}+\cdots+\frac{1}{n}
$$
tiến tới vô cực khi $n \rightarrow+\infty$. Thật vậy,
Xét hàm số $f(x)=\ln (1+x)-x$ với $x>0$. Ta có
$$
f^{\prime}(x)=\frac{1}{1+x}-1<0
$$
nên đây là hàm nghịch biến, suy ra $f(x)<f(0)=0$ hay $\ln (1+x)<$ $x, \forall x>0$. Thay $x$ bởi $\frac{1}{n}$, ta được
$$
\ln \left(1+\frac{1}{n}\right)<\frac{1}{n} \Leftrightarrow \frac{1}{n}>\ln (1+n)-\ln n
$$
Do đó,
$$
\frac{1}{1}+\frac{1}{2}+\frac{1}{3}++\frac{1}{n}>\ln 2-\ln 1+\ln 3-\ln 2+\cdots+\ln (n+1)-\ln n=\ln (n+1)
$$
Vì $\ln (n+1) \rightarrow+\infty$ khi $n \rightarrow+\infty$ nên
$$
\frac{1}{1}+\frac{1}{2}+\frac{1}{3}+\cdots+\frac{1}{n} \rightarrow+\infty
$$
Trở lại bài toán, đặt
$$
y_{n}=\frac{x_{1}+x_{3}+x_{5}+\cdots+x_{2 n-1}}{x_{2}+x_{4}+x_{6}+\cdots+x_{2 n}}
$$
với $n \geq 1$. Ta thấy vì $\frac{1}{n} \in\left(0 ; \frac{\pi}{2}\right)$ nên $\cos \frac{1}{n}>0$, suy ra
$$
x_{n}=\frac{1}{n \cos \frac{1}{n}}>0, n \geq 1
$$

Xét hàm số $f(t)=\frac{t}{\cos t}$ với $t \in\left(0 ; \frac{\pi}{2}\right)$ thì $f^{\prime}(t)=\frac{\cos t+t \sin t}{\cos ^{2} t}>0$ nên đây là hàm đồng biến. Chú ý rằng $x_{n}=f^{2}\left(\frac{1}{n}\right)$, mà $\frac{1}{n}$ là dãy giảm nên $x_{n}$ cũng là dãy giảm.
Suy ra $x_{1}>x_{2}, x_{3}>x_{4}, \ldots, x_{2 n-1}>x_{2 n}$ nên $y_{n}>1$
Ngoài ra, ta cũng có $x_{3}<x_{2}, x_{5}<x_{4}, \ldots, x_{2 n-1}<x_{2 n-2}$ nên
$$
\begin{aligned}
y_{n}<& \frac{x_{1}+\left(x_{2}+x_{4}+\cdots+x_{2 n-2}\right)}{x_{2}+x_{4}+\cdots+x_{2 n}}=\\
& 1-\frac{x_{1}-x_{2 n}}{x_{2}+x_{4}+\cdots+x_{2 n}}<1-\frac{x_{1}}{x_{2}+x_{4}+\cdots+x_{2 n}}
\end{aligned}
$$
Dễ thấy rằng
$$
x_{2}+x_{4}+\cdots+x_{2 n}=\sum_{i=1}^{n} \frac{1}{2 i \cos \frac{1}{2 i}} \geq \sum_{i=1}^{n} \frac{1}{2 i}=\frac{1}{2} \sum_{i=1}^{n} \frac{1}{i}
$$
Theo bổ đề trên thì $\sum_{i=1}^{n} \frac{1}{i}$ tiến tới vô cực nên
$$
\lim \left(x_{2}+x_{4}+\cdots+x_{2 n}\right)=+\infty
$$
Do dó
$$
\lim \left(1-\frac{x_{1}}{x_{2}+x_{4}+\cdots+x_{2 n}}\right)=1-0=1
$$
Theo nguyên lý kẹp, ta có $\lim x_{n}=1$.

Bài 6.

Đặt $X=x-1, Y=y+1$, thay vào, ta có
$$
\begin{aligned}
&\left\{\begin{array}{l}
X^{2}+Y^{2}=b \\
Y-1=(X+1)^{2}+(2 a+1)(X+1)+a^{2}
\end{array}\right. \\
&\Leftrightarrow\left\{\begin{array}{l}
X^{2}+Y^{2}=b \\
Y=X^{2}+(2 a+3) X+a^{2}+2 a+3
\end{array}\right.
\end{aligned}
$$
Ta đưa về tìm điều kiện của $b$ để tồn tại $a$ mà hệ trên có nghiệm $(X, Y)$. Do
$$
Y-(X+2)=X^{2}+2(a+1) X+(a+1)^{2}=(X+a+1)^{2} \geq 0
$$

nên $Y \geq X+2$. Suy ra $Y-X \geq 2>0$, tức là $(X-Y)^{2} \geq 4$. Ta có
$$
b=X^{2}+Y^{2}=\frac{(X-Y)^{2}+(X+Y)^{2}}{2} \geq \frac{(Y-X)^{2}}{2} \geq 2
$$
Mặt khác, với $b \geq 2$, nếu chọn $X=-(a+1)$ thì có $Y=X+2=1-a$. Khi đó, ta có
$$
X^{2}+Y^{2}=(a+1)^{2}+(a-1)^{2}=2\left(a^{2}+1\right)=b
$$
Như thế, với $a$ thỏa mãn $2\left(a^{2}+1\right)=b$ thì hệ có nghiệm là
$$
(X, Y)=(-a-1,1-a)
$$
Dễ dàng thấy rằng do $b \geq 2$ nên luôn tồn tại $a$ như thế.
Vậy các giá trị cần tìm của $b$ là $b \geq 2$.

Bài 7.

1) Giả sử ngược lại, tồn tại 2 cặp $\left(A_{i}, B_{i}\right)$ và $\left(A_{j}, B_{j}\right)$ thỏa mãn điểu kiện đề bài đã cho.
Vì $i \neq j$ nên theo giả thiết,
$$
\left|A_{i} \cap B_{j}\right| \geq 1,\left|A_{j} \cap B_{i}\right| \geq 1
$$
Đặt $x_{r} \in A_{i} \cap B_{j}, x_{s} \in A_{j} \cap B_{i}$ với $1 \leq r, s \leq n$ thì:
– Do $x_{r} \in B_{j}$ nên với mọi $x_{k} \in A_{j}$, ta đều có $k<r$.
– Do $x_{r} \in A_{i}$ nên với mọi $x_{k} \in B_{i}$, ta đều có $k>r$.

Từ đây suy ra
$$
A_{j} \subset\left\{x_{1}, x_{2}, \ldots, x_{r-1}\right\}, B_{i} \subset\left\{x_{r+1}, x_{r+2}, \ldots, x_{n}\right\}
$$
Điều này cho thấy $A_{j} \cap B_{i}=\varnothing$, mâu thuẫn với giả thiết.
Vậy tồn tại không quá 1 cặp $\left(A_{i}, B_{i}\right)$ thỏa mãn điều kiện đã cho.
2) Gọi $T$ là tập hợp các cách chọn hai dãy
$$
A_{1}, A_{2}, \ldots, A_{m} \text { và } B_{1}, B_{2}, \ldots, B_{m}
$$
thỏa mãn điều kiện là: với mỗi $i, j \in\{1,2,3, \ldots, n\}, A_{i} \cap B_{j}=\varnothing$ nếu và chỉ nếu $i=j$.
Gọi $T_{i} \subset T$ là các cách chọn sao cho sao cho cặp $\left(A_{i}, B_{i}\right)$ thỏa mãn điều kiện là: cặp $\left(A_{i}, B_{i}\right)$ với $i=1,2,3, \ldots, n$ sao cho nếu $x_{k} \in A_{i}$ và $x_{l} \in B_{i}$ thì $x_{k}<x_{l}$ (ở đây ta xét thứ tự ban đầu của các phần tử của $X$ ). (*)
Theo câu 1) thì $T_{i} \cap T_{j}=\varnothing$ với $i \neq j$ nên ta có
$$
\left|T_{1}\right|+\left|T_{2}\right|+\cdots+\left|T_{m}\right|=\left|T_{1} \cup T_{2} \cup \ldots \cup T_{m}\right| \leq T
$$
Tiếp theo, với $1 \leq i \leq m$, xét một tập hợp $S \subset X$ và $|S|=a_{i}+b_{i}$. Khi đó, tương ứng với $S$, có đúng 1 cách chọn $\left(A_{i}, B_{i}\right)$ thỏa mãn tính chất $(*)$ – tức là $A_{i}$ sẽ nhận $a_{i}$ số nhỏ nhất trong tập $S, B_{i}$ là lấy phần còn lại.
Trong khi đó, nếu không có điều kiện $(*)$, ta có thể chọn tùy ý $C_{a_{i}+b_{i}}^{a_{i}}$ phần tử trong $S$ và $A$ và số còn lại cho $B$.
Do đó, ta có
$$
\left|T_{i}\right|=\frac{|T|}{C_{a_{i}}^{a_{i}}+b_{i}}
$$
với $i=1,2, \ldots, m$. Từ đây suy ra
$$
\sum_{i=1}^{m} \frac{|T|}{C_{a_{i}+b_{i}}^{a_{i}}} \leq|T| \Leftrightarrow \sum_{i=1}^{m} \frac{1}{C_{a_{i}+b_{i}}^{a_{i}}} \leq 1
$$
Ta có đpcm.

Bài 8.

1) Giả sử $E F$ cắt $B C$ ở $L$ và $(T),(O)$ cắt nhau tại $J$ khác $A$. Suy ra $A J$ chính là trục đẳng phương của $(T),(O)$. Do đó $O T \perp A J$.
Khi đó,
$$
L B \cdot L C=L E \cdot L F
$$
nên $L$ thuộc trục đẳng phương của $(T),(O)$. Suy ra $A, J, L$ thẳng hàng. Theo định lý Brocard cho tứ giác $B E F C$ nội tiếp trong đường tròn $(I)$ thì $I$ chính là trực tâm của tam giác $A D L$.
Vì thế nên ID $\perp A L$, mà $O T \perp A J$ nên $I D \| O T$.

2) Dễ dàng thấy rằng $D$ là trực tâm của tam giác $A G H$ nên $A D \perp G H$. Ta sẽ chứng minh rằng $A, D, K$ thẳng hàng.

Ta có $D B \cdot D F=D E \cdot D C$ nên $D$ có cùng phương tích tới 2 đường tròn $(A B F),(A E C)$. Suy ra $A D$ chính là trục đẳng phương của 2 đường tròn này.

Bằng biến đổi các góc nội tiếp, ta thấy rằng
$$
\angle M P Q=\angle M B F=\angle C E F=\angle C N Q
$$
Suy ra $M N P Q$ nội tiếp, dẫn đến $K M \cdot K P=K N \cdot K Q$, tức là $K$ cũng có cùng phương tích tới 2 đường tròn $(A B F),(A E C)$.
Từ đó suy ra $A, D, K$ thẳng hàng. Do đó, $D K$ vuông góc với $G H$.

Phương trình vô tỉ – Phương pháp nhân chia lượng liên hợp

Phương pháp nhân lượng liên hợp được sự dụng khi phương trình có độ phức tạp cao, lệch bậc nhiều ở các biểu thức chứa căn và nghiệm của phương trình thường dễ đoán và có ít nghiệm.
Nội dung phương pháp là ta phải đoán được nghiệm, thêm bớt (tách) và nhóm các số hạng phù hợp và nhân chia với biểu thức liên hợp để xuất hiện nhân tử. Ta xét các ví dụ sau.
Ví dụ 1
Giải phương trình:
$$\sqrt{3x^2-5x+1}-\sqrt{x^2-2}=\sqrt{3(x^2-x-1)}-\sqrt{x^2-3x+4}$$

Lời giải

Ta có

$\sqrt{3x^2-5x+1}-\sqrt{x^2-2}=\sqrt{3(x^2-x-1)}-\sqrt{x^2-3x+4}$
$\Leftrightarrow \sqrt{3x^2-5x+1}-\sqrt{3(x^2-x-1)}=\sqrt{x^2-2}-\sqrt{x^2-3x+4}$
$\Leftrightarrow \dfrac{-2x+4}{\sqrt{3x^2-5x+1}+\sqrt{3(x^2-x+1)}}=\dfrac{3x-6}{\sqrt{x^2-2}+\sqrt{x^2-3x+4}}$
$\Leftrightarrow
-(x-2)\left[ \dfrac{2}{\sqrt{3x^2-5x+1}+\sqrt{3(x^2-x+1)}}+\dfrac{3}{\sqrt{x^2-2}+\sqrt{x^2-3x+4}}\right] =0$
$\Leftrightarrow x=2.$

(Rõ ràng biểu thức trong ngoặc “[]” là dương)
Thử lại ta thấy $x=2$ thoả mãn.
Vậy $x=2$ là nghiệm của phương trình.

Ta có bước thử lại vì chưa đặt điều kiện của phương trình.

Ví dụ 2 Giải phương trình $$\sqrt[3]{x^2-1}+x=\sqrt{x^3-1}$$

Lời giải
Điều kiện $x \ge \sqrt[3]{2}$.

$\sqrt[3]{x^2-1}-2+x-3=\sqrt{x^2-2}-5$
$\Leftrightarrow (x-3)[1+\dfrac{x+3}{\sqrt[3]{(x^1-1)^2}+2\sqrt[]{x^2-1}+4}]=\dfrac{(x-3)(x^2+3x+9)}{\sqrt{x^3-2}+5}$
$\Leftrightarrow (x-3)[1+\dfrac{x+3}{\sqrt[3]{(x^2-1)^2}+2\sqrt[3]{x^2-1}+4}- \dfrac{x^2+3x+9}{\sqrt{x^3-x}+5}]=0$
$\Leftrightarrow x=3.$

Vì $$1+\dfrac{x+3}{\sqrt[3]{(x^2-1)^2}+2\sqrt[3]{x^2-1}+4}=1+\dfrac{x+2}{(\sqrt[3]{x^2-1}+1)^2+3}<2<\dfrac{x^2+3x+9}{\sqrt{x^3-x}+5}.$$
Vậy phương trình có nghiệm duy nhất $x=3.$

Ví dụ 3 Giải phương trình $\sqrt{x-2}+\sqrt{4-x}=2x^2-5x-1.$

Lời giải
Điều kiện $2 \le x \le 4$.
Khi đó

$\sqrt{x-2}+\sqrt{4-x}=2x^2-5x-1$
$\Leftrightarrow \sqrt{x-2}-1+\sqrt{4-x}-1=2x^2-5x-3$
$\Leftrightarrow \dfrac{x-3}{\sqrt{x-2}+1}-\dfrac{x-3}{\sqrt{4-x}+1}=(x-3)(2x+1)$
$\Leftrightarrow (x-3)[\dfrac{1}{\sqrt{x-2}+1}-\dfrac{1}{\sqrt{4-x}+1}-(2x+1)]=0$
$\Leftrightarrow x=3.$

$\dfrac{1}{\sqrt{x-2}+1} \le 1$
$\dfrac{1}{\sqrt{4-x}+1} \ge \dfrac{1}{\sqrt{2}+1}=\sqrt{2}-1 $
$\Rightarrow \dfrac{1}{\sqrt{x-2}+1}-\dfrac{1}{\sqrt{4-x}+1} \le 2-\sqrt{2}.$
Và $2x+1 \ge 5 $ (do \ x \ge 2$
$
Vậy phương trình có nghiệm duy nhất $x=2.$

Ví dụ 4 Giải phương trình $x^2+x-1=(x+2)\sqrt{x^2-2x+2}$.

Lời giải
Ta có

$x^2+x-1=(x+2)\sqrt{x^2-2x+2}$
$\Leftrightarrow x^2-2x-7+3(x+2)-(x+2)\sqrt{x^2-2x+2}=0$
$\Leftrightarrow x^2-2x-7+(x+2)(3-\sqrt{x^2-2x+2})=0$
$\Leftrightarrow x^2-2x-7-\dfrac{(x+2)(x^2-2x-7)}{\sqrt{x^2-2x+2}+3}=0$
$\Leftrightarrow (x^2-2x-7)(1-\dfrac{x+2}{\sqrt{x^2-2x+2}+3})=0$
$\Leftrightarrow (x^2-2x-7)[\dfrac{\sqrt{(x-1)^2+1}-(x-1)}{\sqrt{x^2-2x+2}+3}]=0$
$\Leftrightarrow x^2-2x-7=0$
$\Leftrightarrow x=1 \pm \sqrt{7}.$
Vậy phương trình có nghiệm $x=1 \pm \sqrt{7}$.

Bài tập rèn luyện

Bài tập 1 Giải các phương trình sau:

a) $\sqrt{2x-3}-\sqrt{x}=2x-6$
b) $\sqrt{x+1}+1=4x^2+\sqrt{3x}$
c) $\sqrt{10x+1}+\sqrt{3x-5}=\sqrt{9x+4}+\sqrt{2x-2}$

d) $\dfrac{2x^2}{(3-\sqrt{9+2x})^2}=x+21$
e) $9(x+1)^2=(3x+7)(1-\sqrt{3x+4})^2$

Bài tập 2 Giải các phương trình sau:

a) $\sqrt{3x+1}-\sqrt{6-x}+3x^2-14x-8=0$
b) $\sqrt{2x^3+3x^2+6x+16}-\sqrt{4-x} =2 \sqrt{3}$
c) $\sqrt{x^2+12}+5=3x+\sqrt{x^2+5}$
d) $x^2-4x-2+\sqrt{x^2-4x+7}+\sqrt{5x-6}=0$
e) $3 \sqrt[3]{x^2}+\sqrt{x^2+8}-2=\sqrt{x^2+15}$

Bài tập 3 Giải các phương trình sau:

a) $\sqrt{2x^2-x+3}-\sqrt{21x-17}+x^2-x=0$
b) $x(x+1)(x-3)+3=\sqrt{4-x}+\sqrt{1+x}$
c) $\sqrt{3x+1}+2\sqrt[3]{19x+8}=2x^2+x+5$
d) $\sqrt{3-x}+\sqrt{2+x}=x^3+x^2-4x-4+|x|+|x-1|$

Bài tập 4 Giải các phương trình sau

a) $\sqrt{x+2}+\sqrt{3-x}=x^3+x^2-4x-1$
b) $3x^2-8x+3=3\sqrt{x+1}$
c) $2x^2-x-2=\sqrt{5x+6}$
d) $\sqrt{x+1}+\sqrt{2x+3}=x^2-x-1$

Đề thi chọn đội tuyển trường Phổ thông Năng khiếu năm học 2020-2021

Ngày thi thứ nhất

Bài 1 . Với mỗi số nguyên dương $n$, tìm số thực $M_{n}$ lớn nhất sao cho với mọi số thực dương $x_{1}, x_{2}, \ldots, x_{n}$ thì ta đều có
$$
\sum_{k=1}^{n} \frac{1}{x_{k}^{2}}+\frac{1}{\left(\sum_{k=1}^{n} x_{k}\right)^{2}} \geq M_{n}\left(\sum_{k=1}^{n} \frac{1}{x_{k}}+\frac{1}{\sum_{k=1}^{n} x_{k}}\right)^{2}
$$
Bài 2. Cho 2021 số nguyên khác 0 . Biết rằng tổng của một số bất kỳ trong chúng với tích của tất cả 2020 số còn lại luôn âm.
(a) Chứng minh rằng với mọi cách chia 2021 số này thành hai nhóm và nhân các số cùng nhóm lại với nhau thì tổng của hai tích cũng luôn âm.
(b) Một bộ số thỏa mãn đề bài thì có thể có nhiều nhất mấy số âm?

Bài 3. Cho hai hàm số $f: \mathbb{R} \rightarrow \mathbb{R}$ và $g: \mathbb{R} \rightarrow \mathbb{R}$ thỏa mãn $g(2020)>0$ và với mọi
$$
x, y \in \mathbb{R} \text { thì }\left\{\begin{array}{l}
f(x-g(y))=f(-x+2 g(y))+x g(y)-6 \
g(y)=g(2 f(x)-y)
\end{array}\right.
$$
(a) Chứng minh rằng $g$ là hàm hằng.
(b) Chứng minh rằng đồ thị $h(x)=f(x)-x$ nhận $x=1$ là trục đối xứng.

Bài 4 . Cho tam giác $A B C$ nhọn, nội tiếp trong đường tròn $(O)$ có trực tâm $H$ và $A H, B H, C H$ cắt cạnh đối diện lần lượt tại $D, E, F$. Gọi $I, M, N$ lần lượt là trung điểm các cạnh $B C, H B, H C$ và $B H, C H$ cắt lại $(O)$ theo thứ tự tại các điểm $L, K$. Giả sử $K L$ cắt $M N$ ở $G$.
(a) Trên $E F$, lấy điểm $T$ sao cho $A T$ vuông góc với $H I$. Chứng minh rằng $G T$ vuông góc với $O H$.
(b) Gọi $P, Q$ lần lượt là giao điểm của $D E, D F$ và $M N$. Gọi $S$ là giao điểm của $B Q, C P$. Chứng minh rằng $H S$ đi qua trung điểm của $E F$.

 

Ngày thi thứ hai

Bài 5. Cho số nguyên dương $n>1$. Chứng minh rằng với mọi số thực $a \in\left(0 ; \frac{1}{n}\right)$ và mọi đa thức $P(x)$ có bậc $2 n-1$ thỏa mãn điều kiện $P(0)=P(1)=0$, luôn tồn tại các số thực $x_{1}, x_{2}$ thuộc $[0 ; 1]$ sao cho $P\left(x_{1}\right)=P\left(x_{2}\right)$ và $x_{2}-x_{1}=a$

Bài 6. Giải phương trình sau trên $\mathbb{Z}^{+}:\left(x^{2}+3\right)^{3^{x+1}}\left[\left(x^{2}+3\right)^{3^{x+1}}+1\right]+x^{2}+y=x^{2} y$.

Bài 7. Cho các số nguyên $n>k>t>0$ và $X={1,2, \ldots, n}$. Gọi $\mathcal{F}$ là họ các tập con có $k$ phần tử của tập hợp $X$ sao cho với mọi $F, F^{\prime} \in \mathcal{F}$ thì $\left|F \cap F^{\prime}\right| \geq t$ Giả sử không có tập con có $t$ phần tử nào chứa trong tất cả các tập $F \in \mathcal{F}$.
(a) Chứng minh rằng tồn tại một tập hợp $B \subset X$ sao cho $|B|<3 k$ và $|B \cap F| \geq t+1$ với mọi $F \in \mathcal{F}$.
(b) Chứng minh rằng $|\mathcal{F}|<C_{3 k}^{t+1} C_{n}^{k-t-1}$.

Bài 8. Cho tam giác $A B C$ nội tiếp trong $(O)$ với $B, C$ cố định và $A$ thay đổi trên cung lớn $B C$. Dựng hình bình hành $A B D C$ và $A D$ cắt lại $(B C D)$ ở $K$.
(a) Gọi $R_{1}, R_{2}$ lần lượt là bán kính đường tròn ngoại tiếp $(K A B),(K A C)$. Chứng minh rằng tích $R_{1} R_{2}$ không đổi.
(b) Ký hiệu $(T),\left(T^{\prime}\right)$ lần lượt là các đường tròn cùng đi qua $K$, tiếp xúc với $B D$ ở $B$ và tiếp xúc với $C D$ ở $C$. Giả sử $(T),\left(T^{\prime}\right)$ cắt nhau ở $L \neq K$ Chứng minh rằng $A L$ luôn đi qua một điểm cố định.

Hết

Lời giải

Đề thi chọn đội Dự Tuyển PTNK năm học 2020-2021

Kì thi chọn Dự tuyển trường Phổ thông Năng khiếu tham dự kì thi 30/04 được tổ chức vào tháng 01 năm 2021, đề gồm 4 bài, làm trong 120 phút.

Đề bài

Bài 1. Cho các số thực không âm $a, b, c$ thỏa mãn $a^{2}+b^{2}+c^{2}=1$. Tìm giá trị lớn nhất của biểu thức $$P=\frac{7}{2} a+(1-a)(\sqrt{a}+\sqrt{b}+\sqrt{c})+a^{2} b^{2} c^{2}$$

Bài 2. Tìm tất cả các hàm số $f: \mathbb{R} \rightarrow \mathbb{R}$ thỏa mãn $f(x-f(y))=4 f(x)+3 x+f(y)$ với mọi $x, y \in \mathbb{R}$.

Bài 3. Cho $n$ là số nguyên dương và $A=\left\{m \in \mathbb{N}^{*} \mid \operatorname{gcd}(m, 6)=1, m<30 n\right\}$ với $|A|=8 n+1$. Chứng minh rằng tồn tại 2 số phân biệt $a, b \in A$ sao cho $a \mid b$.
Bài 4. Cho điểm $M$ di động trên đường thẳng $d$ cố định và $O$ là điểm cố định nằm ngoài đường thẳng $d$. Gọi $A$ là hình chiếu của $O$ lên $d$, và $H$ là hình chiếu của $A$ trên $O M$. Gọi $D$ là trung điểm $H M$.
(a) Chứng minh rằng đường thẳng qua $H$, vuông góc với $A D$ luôn đi qua một điểm cố định. Gọi điểm đó là $N$.
(b) Chứng minh rằng tâm đường tròn $(H M N)$ luôn thuộc một đường thẳng cố định. Từ đó tính tỷ số $\frac{A M}{A O}$ để $(H M N)$ và $(O A H)$ tiếp xúc với nhau.

Ánh xạ – Bài tập

Bài giảng ánh xạ

Bài 1 Trong các quy tắc sau, quy tắc nào là ánh xạ?

a) Xét quy tắc $f$ từ tập các số nguyên $\mathbb{Z}$ vào $X = \{-1, 0 , 1\}$ sao cho với mỗi $x\in \mathbb{Z}$ thì:
$f\left( x \right) = \left\{ \begin{gathered}
– 1 \,\, khi\,\,\,x < 0 \hfill \\
0 \,\, khi\,\,\,x = 0 \hfill \\
1 \,\, khi\,\,\,x > 0 \hfill \\
\end{gathered} \right.$

a)Xét quy tắc cho tương ứng mỗi số thực dương $x$ với số thực $y$ sao cho $y^2 = x$.
b)Cho tương ứng các điểm $M$ thuộc mặt phẳng với các điểm $M’$ thuộc mặt phẳng sao cho $\overrightarrow{MM’} = \overrightarrow{u}$ cho trước.
c)Trong mặt phẳng cho tương ứng điểm $M$ với điểm $M’$ sao cho $MM’ = r > 0$ cho trước.
d)Trong mặt phẳng cho đường thẳng $d$. Quy tắc cho tương ứng $M$ thuộc $d$ ứng với $M$, $M$ không thuộc $d$ ứng với $M’$ sao cho $MM’ \bot d$.
e)Quy tắc cho tương ứng mỗi số hữu tỷ ứng với 1, mỗi số vô tỷ ứng với 0.

Bài 2 Trong các ánh xạ ở bài trên, ánh xạ nào là đơn ánh, song ánh, toàn ánh?

Bài 3 Trong các ánh xạ sau, ánh xạ nào là đơn ánh, toàn ánh, song ánh?

a)Ánh xạ $f: \mathbb{R} \to \mathbb{R}$ thỏa $f(x) = x^3$.
b)Ánh xạ $f: \mathbb{Z} \to \mathbb{N}$ thỏa $f(x) = |x|$.
c)Cho tương ứng mỗi số thực với phần nguyên của nó.

Bài 4 Cho ánh xạ $f: \mathbb{R} \to \mathbb{R}: f(x) = x^2+3x+1$.

a)$f$ có là đơn ánh?
b)$f$ có là toàn ánh không?

Bài 5 Cho $f: (0;1) \to (0;+\infty) $ thỏa $f(x) = \dfrac{x}{1-x}$.

a)Tìm $f(f(x))$.
b)Chứng minh $f$ là song ánh.
c)Tìm ánh xạ ngược của $f$.

Bài 6 Cho $A, B, C, D$ là các tập con của $X$. Đặt ${\chi _D}\left( x \right) = \left\{ \begin{gathered}
1\,\,\,\,\,khi\,\,\,x \in D \hfill \\
0\,\,\,\,khi\,\,\,x \notin D \hfill \\
\end{gathered} \right.$.
Chứng minh rằng:

a)Quy tắc trên là ánh xạ từ $X$ vào ${0, 1}$.
b)$\chi A\cdot \chi _A = \chi_A,\chi{X\backslash A} = 1 – \chi_A$
c)$\chi {A \cap B} = \chi_A.\chi _B,\chi{A \cup B} = \chi_A+ \chi_B – \chi_A\cdot \chi_B$
d)$\chi_A \geqslant \chi _B \Leftrightarrow B \subset A,\chi_A \equiv 0 \Leftrightarrow A = \emptyset $

Bài 7 Cho $f: X \to Y$. $A, B$ là các tập con của $X$; $C, D$ là các tập con của $Y$. Đặt $f(A) = {f(x)|x \in A}$ là tập ảnh của $A$; $f^{-1}(C) = {x \in X|f(x) \in X}$ là tạo ảnh của $C$.

a)Chứng minh nếu $A \subset B$ thì $f(A) \subset f(B)$.
b)Nếu $C \subset D$ thì $f^{-1}(C) \subset f^{-1}(D)$.
c)$f(A\cup B) = f(A) \cup f(B)$.
c)$f(A \cap B) \subset f(A) \cap f(B)$. Và $f(A \cap B) = f(A) \cap f(b)$ khi $f$ là đơn ánh.
d)$f^{-1}(C \cap D) = f^{-1}(C) \cap f^{-1}(D)$ và $f^{-1}(C \cup D) = f^{-1}(C) \cup f^{-1}(D)$.
e)$A \subset f^{-1}(f(A))$.

Bài 8 Cho $h: A \to B$, $g:B \to C$ và $f: C \to D$.

a)Chứng minh rằng nếu $f\circ g$ là đơn ánh và $f$ toàn ánh thì $g$ đơn ánh.
b)Nếu $f \circ g$ là toàn ánh thì $f$ cũng là toàn ánh.
c)Nếu $f, g$ là đơn ánh(toàn ánh, song ánh) thì $f \circ g$ cũng là đơn ánh (toàn ánh, song ánh).
d)Nếu $h$ là song ánh thì $h^{-1}$ cũng là song ánh.
e)Nếu $f \circ g$ và $g \circ h$ là song ánh thì $f, h, g$ cũng là song ánh.

Bài 9 Cho ánh xạ$f:\mathbb{R} \mapsto \left\{ {0,1} \right\}$

$f\left( x \right) = \left\{ \begin{gathered}
1\,\,\,khi\,\,x \in \mathbb{Q} \hfill \\
0\,\,khi\,\,x \notin \mathbb{Q} \hfill \\
\end{gathered} \right.$

a) Tìm tập ảnh của $f$.
b)Tìm ${f^{ – 1}}\left( 1 \right),{f^{ – 1}}\left( 0 \right)$
c)$f$ có là song ánh không? Vì sao?

Bài 10 Cho $A$ và $B$ là hai tập hợp sao cho có một đơn ánh từ $A$ vào $B$. Chứng minh rằng có một toàn ánh từ $B$ vào $A$.

Bài 11 Cho $A$ và $B$ là hai tập hợp sao cho có một toàn ánh từ $A$ vào $B$. Chứng minh rằng có một đơn ánh từ $B$ vào $A$.

Bài 12 Tìm một song ánh từ tập tập các số tự nhiên chẵn đến tập các số tự nhiên lẻ.

Bài 13 Tìm một đơn ánh từ tập các số tự nhiên đến tập các số nguyên.

Bài 14 Tìm một song ánh từ tập các số tự nhiên đến tập các số nguyên.

Bài 15 Tìm một song ánh từ tập $\mathbb{N} \times \mathbb{N}$ đến $\mathbb{N}^*$.

Bài 16 Gọi tập X là tập gồm các khoảng có dạng $\left( {a,b} \right)$ thỏa $0 \leqslant a < b \leqslant 1$.
Xét ánh xạ $X \to \left( {0,1} \right),f\left( {\left( {a,b} \right)} \right) = \frac{{a + b}}{2}$

a)$f$ có phải đơn ánh không? Vì sao?
b)$f$ có phải toàn ánh không? Vì sao?

Bài 17 Cho $X$ là tập khác rỗng, $P(X)$ là tập tất cả các tập con của $X$. Có tồn tại hay không một song ánh đi từ $X$ đến $P(X)$?

Bài 18 Tìm một song ánh từ tập $(0;1)$ đến tập các số thực.

Bài 19 Cho $m$ là số nguyên dương và tập $X = \{-m, -m+1, …, -1, 0, 1, …,m\}$. \Ánh xạ $f: X \to X$ thỏa $f(f(n)) = -n$ với mọi $n \in X$.\
Chứng minh $m$ là số chẵn.

Trắc nghiệm lớp 11 – Đại số – Học kì 1

Chương 1. Hàm số lượng giác – Phương trình lượng giác

Bài 1. Hàm số lượng giác

[WpProQuiz 73]

Bài 2. Phương trình lượng giác cơ bản

Bài 3. Phương trình lượng giác không mẫu mực

Bài 4. Ôn tập chương

Chương 2. Tổ hợp – Xác suất

 

Bài 1. Quy tắc cộng – Quy tắc nhân

Bài 2. Chỉnh hợp  – Hoán vị – Tổ hợp

Bài 3. Nhị thức Newton

Bài 4. Xác suất – Các quy tắc xác suất

Bài 5. Ôn tập chương

[WpProQuiz 20]

Chương 3. Dãy số – Cấp số

Bài 1. Dãy số – Tính chất của dãy số

Bài 2. Cấp số cộng

Bài 3. Cấp số nhân

Bài 4. Ôn tập chương

Bài tập Tập hợp

Lí thuyết

Bài 1. Cho các tập $A, B, C, A’, B’, C’$ là tập con của $X$ thỏa:
a) $A \cup B \cup C = X$;
b) $A \cap B = A’ \cap B’, A \cap C = A’ \cap C’, B \cap C = B’ \cap C’$.
c) $A \subset A’, B\subset B’, C\subset C’$.

Chứng minh $A= A’, B = B’, C = C’$.

Bài 2. Cho $A, B$ là các tập con của $X$, ta kí hiệu đối xứng $A \triangle B = (A \cap (X \setminus B)) \cup (B \cap (X \setminus A))$. Chứng minh rằng:
a) $A \triangle \emptyset = A$.
b) $A \triangle A = \emptyset$.
c) $A \triangle X = X \setminus A$.

Bài 3. Cho tập hợp $E$, $P$ là một phân hoạch của $E$, $\mathscr{A}$ là một bộ phận của $P$. Đặt $F = \{x\in E|\exists A\in \mathscr{A},x\in A\}$. Chứng minh $\mathscr{A}$ là một phân hoạch của $F$.

Bài 4. Cho $E$ là một tập hợp, $n\in \mathbb{N}^*$, $A_o, A_1, \cdots, A_n$ là tập con của $E$ sao cho $$\emptyset \subsetneq A_o \subsetneq A_1 \subsetneq A_2 \subsetneq \cdots \subsetneq A_n = E$$
Đặt $B_o = A_o, B_1 = A_1 \setminus A_o, B_n = A_n \setminus A_{n-1}$.
Chứng minh $B_o, B_1, B_2, \cdots, B_n$ là một phân hoạch của $E$.

Bài 5. Cho $X = \{1, 2, \cdots, n\}$. Cho $F$ là một họ các tập con của $X$, mỗi tập có $r$ phần tử sao cho bất kì $r+1$ tập nào thuộc $F$ thì giao khác rỗng. Chứng minh rằng giao của tất cả các tập trong $F$ cũng khác rỗng.

Bài 6. Cho $A$ là tập con của tập các số hữu tỷ dương thỏa:
a) $1 \in A$.
b) Nếu $x \in A$ thì $x +1 \in A$.
c) Nếu $x \in A$ thì $\dfrac{1}{x} \in A$.
Chứng minh $A$ là tập các số hữu tỷ dương.

Bài 7. Một tập hợp hữu hạn có ít nhất 3 số nguyên dương phân biệt được gọi là tập cân nếu bỏ đi một phần tử bất kì thì các số còn lại có thể chia thành hai tập hợp mà tổng các số trong hai tập hợp đó bằng nhau. Tìm số phần tử nhỏ nhất của một tập cân.

Bài 8.  Cho các số thực $x, y, z$ khác 0 thỏa $xy, yz, xz$ là các số hữu tỉ.
a) Chứng minh $x^2 + y^2 + z^2 $ là số hữu tỉ.
b) Giả sử $x^3+y^3+z^3$ cũng là số hữu tỉ. Chứng minh $x, y, z$ là các số hữu tỉ.

Bài 9. Tìm tất cả các bộ số hữu tỉ dương $(x, y, z)$ sao cho $x+\dfrac{1}{y}, y + \dfrac{1}{z}, z+\dfrac{1}{x}$ là các số nguyên.

Bài 10. Tìm các tập con $A$ khác rỗng của tập ${2,3,4,5,6,…}$ sao cho với mọi $n \in A$ thì cả $n^2+4$ và $\lfloor \sqrt{n} \rfloor +1$ cũng thuộc $A$.

Bài 11. Giả sử tập các số tự nhiên được phân hoạch thành hai tập $A$ và $B$. Chứng minh rằng với mọi số tự nhiên $n$ tồn tại $a, b$ sao cho $a, b, a+b \in A$ hoặc $a, b, a+b \in B$.

Bài 12. Tập hợp $M$ chứa 4 số nguyên phân biệt được gọi là tập liên kết nếu với mỗi $x \in M$ thì ít nhất một trong hai số $x-1, x+1$ thuộc $M$. Gọi $U_n$ là số tập con liên kết của tập $\{1,2,…,n\}$ .

a) Tính $U_7$.
b) Xác định giá trị nhỏ nhất của $n$ sao cho $U_n \ge 2019.$